0
$\begingroup$

Given a set of m cards that have values pairwise different with range 1 to m, what is the probability that after shuffling the card, and picking two of them, the first one is larger than the second one?

This is part of a larger problem I'm trying to solve.

$\endgroup$
2
  • $\begingroup$ What does"values pairwise different" mean? $\endgroup$ Jan 2, 2014 at 13:11
  • $\begingroup$ Means that the cards are unique (no duplicates). $\endgroup$
    – Lincoln
    Jan 2, 2014 at 13:59

1 Answer 1

5
$\begingroup$

Since it is not possible for them to be equal, the problem is symmetric, i.e. the probability that first > second is the same as when second > first, so it must be each one is exactly $1/2$.

On a different level, you are asking if $X,Y \sim \mathcal{U}[1,2, \ldots, m]$ drawn without replacement, what is $\mathbb{P}[X > Y]$? The easiest way is to enumerate the possible drawings. There are exactly $m(m-1)$ choices for the values of the pair $(X,Y)$. Alternatively, note that if $X = k$ then there are $k-1$ choices for $Y$ so that $X > Y$. So the total number of choices where $X>Y$ is $$ \sum_{k=1}^m (k-1) = \sum_{k=0}^{m-1} k = \frac{m(m-1)}{2}. $$ Thus, the probability of $X>Y$ is $\dfrac{m(m-1)/2}{m(m-1)} = \dfrac{1}{2}$.

$\endgroup$
3
  • $\begingroup$ Hmm that's interesting. The original problem is based on picking all the cards one after the other, and placing cards that are smaller than the largest picked number yet on one pile, and the others on the other pile. Given the explanation you gave above for picking just 2 of the cards, does that mean that the rest of the solution would give an m(m-n)/2 / m(m-n), for n between 0 and m-1 ? $\endgroup$
    – Lincoln
    Jan 2, 2014 at 14:01
  • $\begingroup$ @grixtil I think you should post your more general problem as a separate question -- if you'd like it answered here. The answer above really does say everything there is to say about this question. $\endgroup$
    – Carl
    Jan 2, 2014 at 14:17
  • $\begingroup$ Thanks @Carl I've done that at this other post $\endgroup$
    – Lincoln
    Jan 2, 2014 at 15:09

You must log in to answer this question.

Not the answer you're looking for? Browse other questions tagged .